LSAT and Law School Admissions Forum

Get expert LSAT preparation and law school admissions advice from PowerScore Test Preparation.

 Administrator
PowerScore Staff
  • PowerScore Staff
  • Posts: 8917
  • Joined: Feb 02, 2011
|
#40926
Complete Question Explanation
(The complete setup for this game can be found here: lsat/viewtopic.php?t=1864)

The correct answer choice is (B)

Again, prior work can be used to eliminate all of the incorrect answer choices:

Question #17. The solution in the correct answer shows that L and N can be used consecutively, which eliminates answer choice (A). This solution also shows that T and V can be used consecutively, which eliminates answer choice (D).

Question #19. The solution presented to answer choice (D) of this question shows that T and W can be used consecutively, which eliminates answer choice (E). This same solution can also be used to again eliminate answer choice (A).

Question #21. The solution presented in this question shows that N and T can be used consecutively (T used fourth, N used fifth), which eliminates answer choice (C). The same solution also shows that T and W can be used consecutively (W used third, T used fourth), which again eliminates answer choice (E).

Thus, every answer choice is eliminated except answer choice (B), which is the correct answer.

Get the most out of your LSAT Prep Plus subscription.

Analyze and track your performance with our Testing and Analytics Package.